Quick question on exponential decay problem

Click For Summary
The discussion revolves around a misunderstanding of an exponential decay problem's final answer. The original poster's answer differs from the correct solution due to a potential oversight in calculating the definite integral, specifically neglecting the lower bound. Participants suggest reviewing the integration steps and emphasize the importance of including the lower limit in the calculations. Clarification is provided regarding the subtraction of the exponential term, which becomes clearer upon considering the value at the lower limit of integration. Ultimately, the issue was resolved by recognizing the missed term in the calculations.
ozone
Messages
121
Reaction score
0
I'm attaching the problem as a png. The top half is the question whereas the second half is the solution. I understand everything about the question until the ultimate answer

the final answer is: (r (constant) x0(constant) / k (constant)) * (1 - e^-60t)
as shown.

However I don't understand why my answer differs. I concluded the problem with
(r (constant) x0(constant) / k (constant)) * (-e^-60t)
This doesn't seem to be a basic algebra issue, and It is beyond my comprehension.

Thank you.
 

Attachments

  • problem&solution.png
    problem&solution.png
    32.5 KB · Views: 583
Physics news on Phys.org
Did you have the same integral as they did? If so, just looking at the last step where they calculate the definite integral, you must have made an algebra error. If that's not the problem, can you post your work so we can look for where you made a mistake?

Perhaps you forgot the term with the 0 (the lower bound) when computing your definite integral?
 
I guess it could just be algebraic. I didn't quite understand why it was that they were taking 1 minus the e term. Could you explain please?
 
What is the value of the exponential when the lower limit of integration is taken (t = 0)?
 
oh wow now I see.. Thanks for helping me to see the obvious!
(I ignored the lower bound~)
 
Question: A clock's minute hand has length 4 and its hour hand has length 3. What is the distance between the tips at the moment when it is increasing most rapidly?(Putnam Exam Question) Answer: Making assumption that both the hands moves at constant angular velocities, the answer is ## \sqrt{7} .## But don't you think this assumption is somewhat doubtful and wrong?

Similar threads

  • · Replies 7 ·
Replies
7
Views
3K
Replies
1
Views
8K
  • · Replies 18 ·
Replies
18
Views
4K
Replies
20
Views
2K
  • · Replies 4 ·
Replies
4
Views
2K
  • · Replies 1 ·
Replies
1
Views
1K
  • · Replies 1 ·
Replies
1
Views
4K
  • · Replies 7 ·
Replies
7
Views
2K
  • · Replies 3 ·
Replies
3
Views
5K
  • · Replies 7 ·
Replies
7
Views
2K